Line 31: Line 31:
 
for N larger or equal to M, <math>\left| \alpha\right| < 1</math>, the equation above holds.
 
for N larger or equal to M, <math>\left| \alpha\right| < 1</math>, the equation above holds.
  
//Did I make any mistake in the N+1 part or it's just a typo?
 
  
 +
//Did I make any mistake in the N+1 part?
 +
 +
//ahhh, the series is finite, so the condition <math>\left| \alpha\right| < 1</math> doesn't  necessarily hold. but it requires an other approach with
 +
 +
<math> \sum_{n=-\infty}^{0} \alpha^n = \frac{1}{(1 - \frac{1}{\alpha})} , \left| \alpha \right|  > 1</math>;     
 +
 +
Anyone want to finish it for me? Thanks
  
 
===Answer 2===
 
===Answer 2===

Revision as of 09:44, 10 September 2011

When is this super duper geometric series formula valid?

A student in ECE301 once wrote the following formula on his exam:

$ \sum_{n = M}^N \alpha^n = \frac{\alpha^M - \alpha^{N-1}}{(1 - \alpha)} $

Is this formula correct? For what values of the parameters is the formula valid? Please comment.


Share your answers below

You will receive feedback from your instructor and TA directly on this page. Other students are welcome to comment/discuss/point out mistakes/ask questions too!


Answer 1

First we know the summation of an infinity geometric series: $ \sum_{n=0}^{\infty} \alpha^n = \frac{1}{(1 - \alpha)} , \left| \alpha \right| < 1 $; (eq1)

so we can compute

$ \sum_{n=M}^{\infty} \alpha^n = \left( \alpha \right)^M \frac{1}{(1 - \alpha)} , \left| \alpha \right| < 1 $; (eq2)

similarly,

$ \sum_{n=N+1}^{\infty} \alpha^n = \left( \alpha \right)^{N+1} \frac{1}{(1 - \alpha)} , \left| \alpha \right| < 1 $; (eq3)

then we can substract eq3 from eq2, if N+1> M

$ \sum_{n=M}^{\infty} \alpha^n - \sum_{n=N+1}^{\infty} \alpha^n = \frac{{\left( \alpha \right)^M } - {\left( \alpha \right)^{N+1}}}{(1 - \alpha)} , \left| \alpha \right| < 1 $;

for N larger or equal to M, $ \left| \alpha\right| < 1 $, the equation above holds.


//Did I make any mistake in the N+1 part?

//ahhh, the series is finite, so the condition $ \left| \alpha\right| < 1 $ doesn't necessarily hold. but it requires an other approach with

$ \sum_{n=-\infty}^{0} \alpha^n = \frac{1}{(1 - \frac{1}{\alpha})} , \left| \alpha \right| > 1 $;

Anyone want to finish it for me? Thanks

Answer 2

Write it here.

Answer 3

Write it here


Back to ECE438 Fall 2011 Prof. Boutin

Back to ECE301

Back to ECE438

More on geometric series

Alumni Liaison

Abstract algebra continues the conceptual developments of linear algebra, on an even grander scale.

Dr. Paul Garrett